Mrs. Khan purchased 6 sets of dress material costing Rs.1235 each and 5 shirts of Rs.1635 each. How much does she spend for dress materials?

Answers

Answer 1

Answer:

Rs. 6175

Step-by-step explanation:

Number of dress materials purchased = 6

Cost of each dress material = Rs.1235

Number of shirts purchased = 5

Cost of each shirt = Rs.1635

How much does she spend for dress materials?

Total cost of dress material = Number of dress materials purchased × Cost of each dress material

= 5 × Rs.1235

= Rs. 6175

Total cost of dress material = Rs. 6175


Related Questions

Arturo is building a flower bed in the shape of a right triangle. The hypotenuse of the right triangle is 13 feet. One of the legs needs to be 7 feet longer than the other leg.

Which equation can be used to find x, the length of the shorter leg?

x(x + 7) = 13
x(x + 7) = 169
x2 + (x + 7)2 = 13
x2 + (x + 7)2 = 169

Answers

Answer:

x(x + 7) = 169

Step-by-step explanation:

We are given that

Let length of shorter leg of right triangle = x ft

Length of other leg = x+7 ft

Hypotenuse of right triangle = 13 ft

We have to find the equation which can be used to find x, the length of shorter leg.

By pythagorous theorem

(Hypotenuse)^2=(Base)^2+(Perpendicular\;side)^2

Substitute the values then, we get

(13)^2=x^2+(x+7)^2

x^2+(x+7)^2=169

This is required equation which can used to find x.

Answer:

d. x2 + (x + 7)2 = 169

Step-by-step explanation:

use Pythagorean theorem, substitute in the given values, and you get this

It took Sarah 4 days to write a paper, she wrote 12 pages on day 1, 15 pages on day 2 and 9 pages on day 3. If she wrote 12 pages per day , how many pages did she write on the fourth day ?

Answers

Answer:

Option A

Step-by-step explanation:

Sarah took 4 days to write a paper.

She wrote 12 pages per day, so total number of pages she wrote in 4 days = 12 × 4

= 48 pages

On day 1, she wrote number of pages = 12

On day 2, she wrote number of pages = 15

On day 3, she wrote number of pages = 9

On day 4, she wrote number of pages = P

She wrote total number pages in 4 days = 12 + 15 + 9 + P

                                                                    = 36 + P

Therefore, P + 36 = 48

P = 48 - 36

P = 12

She wrote 12 pages on day 4.

Option A is the answer.

Cayden has several screws on a scale, and the scale reads 80.955 cayden add 1 more screw and the scale reads 84.81

Answers

Answer:

-3.855

Step-by-step explanation:

Not sure if you need the weight of each screw! but you just subtract 80.955-84.81

Answer:

1

Step-by-step explanation:

itnisbbsndnfnfnfnnfnfnfncjcjccjcj

Find value of x.
A. 110
B. 47
C. 68
D. 112

Answers

Answer:

B

Step-by-step explanation:

The sum of the inner angles of a quadrilateral is 360 degrees

135 + 110 + 68 + x = 360

313 + x = 360

x = 47 degrees

Answer:

47

Step-by-step explanation:

whole thing is 360 degrees

68 + 110 + 135 = 313

360 - 313 = 47

x looks small too (if you had to guess in a multiple choice question)

If a bus travel for 120 minutes at a speed of 75 kilometers per hour how far has the bus traveled?

Answers

Answer:

150 km

Step-by-step explanation:

Put the minutes into hours 120min is 2 hours.

Distance = speed * time

Distance = 75 * 2

Distance = 150

Answer:

150 kilometers

Step-by-step explanation:

if the bus is going 75 kilometers an hour and they traveled for 120 minutes (exactly 2 hours) then you would just multiply 75 by 2 to get 150 kilometers total.

What is the quotient of (x^3 - 3x^2 + 3x - 2) ÷ (x^2 - x + 1)?
O x - 2
O x + 2
O x- 4
O x + 1

Answers

Answer:

x-2

The choose (1)

Step-by-step explanation:

(x³-3x²+3x-2)÷(x²-x+1)

(x-2)(x²-x+1) ÷ (x²-x+1)

Delete (x²-x+1)

so = (x-2)

Hello, Brainly community!

This question is for all of those Calculus people out there.

The volume of a swimming pool is changing with respect to time, such that the volume is given by W(t), where W(t) is measured in cubic centimeters and t is measured in seconds. A tangent line is shown for W(t) at t = 3 seconds. Determine the best estimate for the value of the instantaneous rate of change of W(t) when t = 3.
(I've narrowed down the answer choices to 2, and just really need to find the right way of thinking to find the answer)

(A) W(lim t) as t goes to 3.
(B) [W(3.1) - W(2.9)] / 0.2.

Thank you in advance!

Answers

Answer:

(B)  [tex]\displaystyle \frac{W(3.1) - W(2.9)}{0.2}[/tex]

General Formulas and Concepts:

Calculus

Limits

Derivatives

The definition of a derivative is the slope of the tangent line.

Derivative Notation

Instantaneous Rates

Tangent Line: [tex]\displaystyle f'(x) = \frac{f(b) - f(a)}{b - a}[/tex]

Step-by-step explanation:

Since we are trying to find a rate at which W(t) changes, we must find the derivative at t = 3.

We are given 2 close answer choices that would have the same numerical answer but different meanings:

(A)  [tex]\displaystyle \lim_{t \to 3} W(t)[/tex](B)  [tex]\displaystyle \frac{W(3.1) - W(2.9)}{0.2}[/tex]

If we look at answer choice (A), we see that our units would simply just be volume. It would not have the units of a rate of change. Yes, it may be the closest numerically correct answer, but it does not tell us the rate at which the volume would be changing and it is not a derivative.

If we look at answer choice (B), we see that our units would be cm³/s, and that is most certainly a rate of change. Answer choice (B) is also a derivative at t = 3, and a derivative tells us what rate something is changing.

∴ Answer choice (B) will give us the best estimate for the value of the instantaneous rate of change of W(t) when t = 3.

Topic: AP Calculus AB/BC (Calculus I/I + II)

Unit: Differentiation

Book: College Calculus 10e

The area of the rectangular sandbox at Dave's school is 117 square feet. The sandbox has a width of 9 feet as shown in the diagram. What is the perimeter of the sandbox?

Answers

Answer:

ay bru ima tell yu dhis rn is c

Step-by-step explanation:

someone help me for this algebra task please

Answers

i just need to answer questions

What’s the equation of the line that passes through the point (-4,4) and has a slope of 3/4

Answers

Answer:

y-y1=m(x-x1)

y-4=3/4(x+4)

y=3/4x+7

There are 35 times as many students at Wow University as teachers. When all the students and
teachers are seated in the 8544 seat auditorium, 12 seats are empty. How many students attend
Wow University?

Answers

Given:

There are 35 times as many students at Wow University as teachers.

When all the students and teachers are seated in the 8544 seat auditorium, 12 seats are empty.

To find:

The total number of students.

Solution:

Let x be the number of teachers at Wow University. So, the number of student is :

[tex]35\times x=35x[/tex]

When all the students and teachers are seated in the 8544 seat auditorium, 12 seats are empty.

[tex]x+35x=8544-12[/tex]

[tex]36x=8532[/tex]

[tex]x=\dfrac{8532}{36}[/tex]

[tex]x=237[/tex]

The number of total students is:

[tex]35x=35(237)[/tex]

[tex]35x=8295[/tex]

Therefore, the total number of students is 8295.

pls help me don't know what to do

Answers

Answer:

x=15

Step-by-step explanation:

The 60 degree angle and the (x+45) degree angle are both the same degree because they are vertical angles.

So to solve, just subtract 45 from 60

60-45=15

That's your answer!

Hope this helps!

D=22/7×d-90 Solve the equation
Find D​
Fast!

Answers

Answer:

D=22-90+22d/7

Step-by-step explanation:

D=22/7×d-90

D=-90+22d/7

Answer is- d=-630/-22x+7

The expression 2x and x² have the same value for only two values of x. What are these values?​

Answers

Answer:

0 and 2

Step-by-step explanation:

Evaluate 2y when y = 6y

Answers

I believe the answer is 12y
im sure the answer is 12y

Choose the smallest number 3 1/8 or 10/3

Answers

Answer:

10/3

Step-by-step explanation:

31/8= 3.8

10/3= 3.3

A cinema is doing a promotion to celebrate their 50th anniversary for 1 week. They give

away a free drink to every 98th customer, a free bag of popcorn to every 112th customer and

a free cinema ticket to every 224th customer. Which lucky customer will be the first to

receive all 3 items?​

Answers

Answer:

1,568 customer

Step-by-step explanation:

Find the lowest common multiple of 98, 112, and 224

98 = 98, 196, 294, 392, 490, 588, 686, 784, 882, 980, 1078, 1176, 1274, 1372, 1470, 1568, 1666

112 = 112, 224, 336, 448, 560, 672, 784, 896, 1008, 1120, 1232, 1344, 1456, 1568, 1680, 1792, 1904

224 = 224, 448, 672, 896, 1120, 1344, 1568, 1792, 2016, 2240

The lowest common multiple of 98, 112, and 224 is 1568

Therefore, the 1,568th customer will be the first to receive all 3 iitem

Which operation will solve the following word problem? Jeff earns $14.00 per hour, Tom earns half as much as Jeff. How much does Tom earn per hour?


Multiplication


Subtraction


Addition


Division

Answers

Answer:

The correct option is (d).

Step-by-step explanation:

Given that,

Jeff earns $14.00 per hour.

Tom earns half as much as Jeff.

We need to find the amount earn by Tom per hour.

Tom's amount = Jeff's amount/2

So,

[tex]T=\dfrac{14}{2}\\\\T=\$7[/tex]

So, Tom earn $7 per hour. Hence, division operation is used. Jeff's amount is divided by 2.

Find ∠MPN

Help me please

Answers

Answer:

[tex]22^{\circ}[/tex]

Step-by-step explanation:

Line [tex]\overline{PM}[/tex] is a diameter of the circle because it passes through the circle's center O. Therefore, arc [tex]\widehat{PLM}[/tex] must be 180 degrees, as these are 360 degree in a circle.

We can then find the measure of arc [tex]\widehat{LM}[/tex]:

[tex]\widehat{LP}+\widehat{LM}=180^{\circ},\\92^{\circ}+\widehat{LM}=180^{\circ},\\\widehat{LM}=88^{\circ}[/tex]

Arc [tex]\widehat{LM}[/tex] is formed by angle [tex]\angle LPM[/tex]. Define an inscribed angle by an angle with a point on the circle creating an arc on the circumference of the circle. The measure of an inscribed angle is exactly half of the measure of the arc it forms.

Therefore, the measure of [tex]\angle LPM[/tex] must be:

[tex]m\angle LPM=\frac{88}{2}=44^{\circ}[/tex]

Similarly, the measure of [tex]\angle LNP[/tex] must be:

[tex]m\angle LNP=\frac{92}{2}=46^{\circ}[/tex]

Angles [tex]\angle LPM[/tex] and [tex]\angle MPN[/tex] form angle [tex]\angle LPN[/tex], which is one of the three angles in [tex]\triangle LPN[/tex]. Since the sum of the interior angles of a triangle add up to 180 degrees, we have:

[tex](\angle MPN+\angle LPM)+\angl+ PLN+\angle LNP=180^{\circ},\\\angle MPN+44+46+68=180,\\\angle MPN=180-44-46-68,\\\angle MPN=\boxed{22^{\circ}}[/tex]

A parabola opens upward. The parabola goes through the point (3,-1),
and the vertex is at (2,-2).

Find the value of A for the parabola. Show your work. Use Part 1 and 2 to write the equation of the parabola.

Answers

Answer:

a=1

Step-by-step explanation:

Hopefully this helps :)

The equation of the parabola is: y = (x - 2)² - 2. Finding the value of A

The vertex of the parabola is at (2,-2). Since the parabola opens upward, the equation of the parabola will be of the form:

y = A(x - 2)² - 2

We can plug the point (3,-1) into this equation to find the value of A.

-1 = A(3 - 2)² - 2

Simplifying the right side of the equation, we get:

-1 = A - 2

Adding 2 to both sides of the equation, we get:

1 = A

Therefore, the value of A is 1.

Writing the equation of the parabola

The equation of the parabola is:

y = (x - 2)² - 2

To know more about parabola:

https://brainly.com/question/11911877

#SPJ2

An 80-mile trip is represented on a gridded map by a directed line segment from point M(3, 2) to point N(9,
13). What point represents 60 miles into the trip? Show your work and explain your reasoning.

Answers

Answer:

Step-by-step explanation:

If the distance from M to N is 80 miles and we want to find the coordinates of the point 60 miles into the trip, we are looking for the point 3/4 of the way from M to N, since 60 is 3/4 of 80. This is the process:

First we need to consider that MN is a directed vector. We first find the components of the directed vector, which is found in the change in x and the change in y. First step, then, looks like this:

<Δx, Δy> = <9-3, 13-2> = <6, 11> We will call those the x and y components of the vector (which comes from vector study in both physics and math, so if you don't understand that, it's ok! Just follow the process here and you'll be fine). Knowing that 60 is 3/4 of the way from M to N, we find 3/4 of both the x and y components, like this:

[tex]<\frac{3}{4}(6), \frac{3}{4}(11)>[/tex] which will give us 3/4 of the change in x and 3/4 of the change in y:

[tex]<\frac{18}{4},\frac{33}{4}>=<4.5,8.25>[/tex]

Since we are going 3/4 of the way from M to N, we add that component to the x and y coordinates of M, giving us 3/4 of the way from M to N, which translates to 60 miles of the 80 mile trip:

(3 + 4.5, 2 + 8.25) = (7.5, 10.25)

Those are the coordinates of the point that represents 60 miles into the 80 mile trip.

Greg buys 60 garden plants at a cost price of $2.00 each to sell in his shop. He sells 25 of them at the profit of 75% and 18 of them at the profit of 35%. He sells the rest of the plants for 4/5 of the cost price calculate the profit or loss he makes from selling 60 plants stating if it is a profit or loss

Answers

Answer:

$43.30 profit

Step-by-step explanation:

Total cost of plant:

60*2 = 120

Greg makes total of:

25*(2 + 0.75*2) + 18*(2 + 0.35*2) + (60 - 25 - 18)*2*4/5 = 163.3

Since Greg mare than cost, he has a profit and the amount is:

163.3 - 120 = 43.3

PLS HELP SOON WILL MARK BRAINLYEST

A railroad tunnel is shaped like a semi-ellipse, as shown below. A semiellipse is shown on the coordinate plane with vertices on the x axis and one point of intersection with the positive y axis. The height of the tunnel at the center is 35 ft, and the vertical clearance must be 21 ft at a point 8 ft from the center. Find an equation for the ellipse.

Answers

According to the question

b= 35 and (8,21) lies on the ellipse

After calculation we get a= 10

equation for the ellipse.

[tex] \frac{ {x}^{2} }{100} + \frac{ {y}^{2} }{1225} = 1[/tex]

lidentify the domain of the function shown in the graph
O A 15257
O B. 19334
O C. 221
O D. All real numbers

Answers

Answer:

B.

Step-by-step explanation:

the visible line is the defined function.

this line goes from x=1 to x=4, and has the functional results from y=1 to y=7.

the domain is the valid interval of the input variable (typically x), while the range is the valid inescapable of the result variable (typically y).

so, B is the right answer.

According to the number line, what is the distance between points A and B?

0 6 units
7 units
O 12 units
O 14 units

Answers

Answer:

14 units

Step-by-step explanation:

A = - 2, B = 12

Therefore,

d(A, B) = 12 - (-2) = 12 + 2 = 14 units

A statistics professor asked students in a class their ages. On the basis of this information, the professor states that the average age of all the students in the university is 24 years. This is an example of

Answers

Answer:

propbability ???

Step-bp explanation:

Answer:

Step-by-step explanation:

This is an example of a statistical mean.

The center of the circle is at the point
, and its radius is
units. The equation of this circle in standard form is
.

Answers

Is there a picture that goes with this?

Solve for x

Help me please

Answers

Answer:

x = 42.5

Step-by-step explanation:

A triangle inscribed inside of a circle using the diameter of the circle as it's hypotenuse is a right triangle ( a 90 degree triangle ) where the angle opposite of the hypotenuse is the 90 degree angle ( angle C )

Hence angle C = 90

If angle C = 2x + 5

Then 2x + 5 = 90

Notice that we just created an equation that we can use to solve for x

We now solve for x algebraically

2x + 5 = 90

Step 1 subtract 5 from both sides

2x + 5 - 5 = 90 - 5

Outcome: 2x = 85

Step 2 divide both sides by 2

2x/2 = 85/2

Outcome: x = 42.5

find the domain of f(x)=sec(2x)

Answers

Answer:

*Refer the image attached

Step-by-step explanation:

*Refer the image attached

2 1/4 x 3 1/5 brainliest

Answers

Answer:

36/5

Step-by-step explanation:

9/4×16/5

144/20

36/5

hope this is helpful

Answer:

[tex]7\frac{1}{5}[/tex]

Step-by-step explanation:

1. start by turning the fractions improper fractions:

[tex]2\frac{1}{4} =\frac{9}{4}[/tex]

[tex]3\frac{1}{5} =\frac{16}{5}[/tex]

2. then multiply them together:

[tex]\frac{9}{4}[/tex] x [tex]\frac{16}{5}[/tex] = [tex]\frac{144}{20}[/tex]

3. then simplify the fraction:

[tex]\frac{144}{20}[/tex][tex]=\frac{36}{5}[/tex]

4. turn it into a proper fraction:

[tex]\frac{36}{5} =7\frac{1}{5}[/tex]

Other Questions
Cmo transformamos las fuentes de energa renovable en energa elctrica necesarias para el desarrollo sostenible de nuestro pas? 3. A)Find the next number in the sequence.$1,27, 9, 3, _1_B) Is the sequence arithmetic, geometric, or neither?Help me find this answer please what is the area of this triangle Solve for x. Round to the nearest tenth, if necessary. Calculating Sin or Cos or Tan A balloon at sea level on earth (1 atm pressure, 19C) takes up 14.5 L of space. The balloon travels to Mars where atmospheric pressure is 4.55 torr and the temperature is -55C What is the volume of the balloon on Mars? the regular price of base ball cleats is 80 dollars if the cleats are on sale for 45% off, then what is the value of the discount in dollars? A chemist combines 33 g of methane with 289 g of oxygen to from 189 g of carbon dioxide and 30 g of water. Determine if the results of the following word problem adheres to the Law of Conservation of Mass. Dog breeds 2238 pages in seven hours he reads the same number of pages each hour how many pages did he read in one hour Below are the account balances for Cowboy Law Firm at the end of December. Accounts Balances Cash $ 3,800 Salaries expense 1,400 Accounts payable 1,800 Retained earnings 4,700 Utilities expense 1,200 Supplies 12,200 Service revenue 7,700 Common stock 4,400 Required: Use only the appropriate accounts to prepare an income statement. Write an equation that represents the line.Use exact numbers. The extinction vortex represents the idea that even if an organism is extant, it may have a gene pool that will not support its long-term survival.A. TrueB. False Suppose that X1 and X2 are independent random variables each with a mean and a variance ^2. Compute the mean and variance of Y = 3X1 + X2. What number is increased by 40% become 28?a.5 b. 10 c. 15 d. 20 Read the student thesis. Parents should, if they choose, limit the amount of television their children are viewing. The best way to improve this thesis is to include the pros and cons of the topic. provide a clearly expressed viewpoint. be neutral and refrain from taking a side. agree with and support any counterclaims. What are the advantages of using genetic testing to aid in drug selection for patients? Select all correct answers from the list below.Check All That ApplyA) Patients likely to experience adverse reactions to a drug can be identified.B) When several drugs are available for the same condition, the drug most likely to be effective can be selected.C) Allergic reactions to drugs can be prevented.D) The response to drug treatment can be monitored.E) The patients prognosis can be predicted worksheet 64editing (errors)The following passages have not been edited. There is an error in each line with a blank against it. write the incorrect word and the correct word in the space provided against each line. The first one has been done for you. incorrect correct 4. I step out of the hostel (a) step stepped gate. Our hostel was not much (b)......... .........than a hundred yard from the (c)........ ........ river. The sand was damp with a (d)....... .......morning dew. But as I bury (e)....... ....... my feet, I felt the warmth for (f)........ ...... the previous day's sun. There was immense (g) ........ ........ banyan trees and birds stirred and chirped on their nests. (h) ........... ........... I felt I am really in a new world. (i)......... ........... Which of the following slopes of a line pass through points (3, 1) and (0, 1)? Triangle Angle-Sum Theorem In a short paragraph, describe the importance of Hollywoods contributions during the war. Be sure to use at least two examples from your assignment. Given EH = 12 and EG = 20 , find FJ.